Pharm Exam 4

¡Supera tus tareas y exámenes ahora con Quizwiz!

Vascular phase of inflammation

Occurs 10-15 min after injury; Associates with vasodilation, relaxation of smooth muscle, increased capillary permeability and sensitization of nerve cells to pain

The nurse is caring for a patient treated with flumazenil (Anexate) for benzodiazepine toxicity. After administering flumazenil what will the nurse carefully assess for? A) Agitation, confusion, and seizures B) Cerebral hemorrhage and dystonia C) Hypertension and renal insufficiency D) Hypotension, dysrhythmias & cardiac arrest

A ~ Administration of flumazenil blocks the action of benzodiazepines. If the patient has been taking these medications for an extended period of time, the blockage of the drugs effects could precipitate an acute benzodiazepine withdrawal syndrome with symptoms including agitation, confusion, and seizures. Anexate does not cause cerebral hemorrhage and dystonia, hypertension, renal insufficiency, hypotension, dysrhythmias, and cardiac arrest.

Extrapyramidal symptoms are a side effect of perphenazine (Trilafon). The nurse should assess and observe for which sign of akathisia? a. Restlessness and constant moving about b. Facial grimacing c. Chewing motion d. Involuntary eye movement

A ~ Akathisia is described as restless, agitated movement.

The nurse administers promethazine (Phenergan) to the patient before sending the patient to the preoperative holding area. What is the rationale for administration of this drug? A) Sedation B) Oral secretions C) Hypotension and bradycardia D) Confusion

A ~ Antihistamines (promethazine, diphenhydramine [Benadryl]) can be very sedating in some people. They are used as preoperative medications and postoperatively to decrease the need for narcotics. Promethazine is not given for hypotension, bradycardia, confusion, or oral secretions.

A patient explains to a nurse that he had been taking amitriptyline (Elavil) for depression and that his physician changed his medication to clomipramine (Anafranil). The patient is confused and does not understand why his medication was changed. The nurses best response to the patient would be what? A) These drugs are similar but some patients respond better to one drug than another. B) Did you take the amitriptyline like you should have? C) Maybe the old medicine wasnt working anymore. D) Clomipramine is newer and will be much better for you.

A ~ Because all tricyclic antidepressants (TCAs) are similarly effective, the choice of which TCA depends on individual response to the drug and tolerance of adverse effects. A patient who does not respond to one TCA may respond to another drug from this class. In addition, the nurse might inform the physician of the patients question so the physician can explain his or her rationale for changing medications. By asking the patient if he took the medication as prescribed, the nurse is insinuating that he may not have and could damage the trusting nursepatient relationship. The nurse has no basis for commenting that the medication might not be working or that another drug would work better.

Obsessive-compulsive disorder (OCD) is a disorder that remains under investigation as to its actual neurophysiology. What tricyclic antidepressant is now approved by the Food and Drug Administration to treat OCD? A) Clomipramine B) Imipramine C) Nortriptyline D) Amitriptyline

A ~ Clomipramine is now also approved for use in the treatment of OCD. Imipramine, nortriptyline, and amitriptyline are not approved for use in treating OCD.

A 75-year-old patient is brought to the emergency department by his family. The family relates that the patient is complaining of confusion, seizures, and abnormal perception of movement. The nurse reviews all of the medication bottles found in the house and suspects the patient overdosed on what medication? A) Benzodiazepine B) Antihypertensive C) Sedative D) Analgesic

A ~ Common manifestations of benzodiazepine toxicity include increased anxiety, psychomotor agitation, insomnia, irritability, headache, tremor, and palpitations. Less common but more serious manifestations include confusion, abnormal perception of movement, depersonalization, psychosis, and seizures. These symptoms are not found in association with options B, C, or D.

A client is taking an anxiolytic agent secondary to grief-related anxiety. The client questions the nurse about abruptly discontinuing these agents. The nurses response is based on the knowledge that, when discontinuing these medications: a. the dosage must be tapered to avoid withdrawal. b. the client must be evaluated for hyperglycemia. c. hangover syndrome must be planned for. d. blood levels must be monitored.

A ~ Discontinuing anxiolytic agents abruptly may lead to withdrawal symptoms.

A client is ordered to receive chlordiazepoxide (Librium) for severe anxiety. The nurse monitors for which symptoms of severe anxiety or panic attack? a. Dyspnea and heart palpitations b. Trembling, shaking, and gastrointestinal upset c. Dizziness and anorexia d. Drowsiness and blurred vision

A ~ Dyspnea and heart palpitations are symptoms of severe anxiety; also experienced is chest pain, dizziness, or faintness.

A patient is admitted to the unit with obsessive-compulsive disorder (OCD). What drug might the nurse administer that has been found to be effective for treating OCD? A) Fluvoxamine B) Phenelzine C) Desipramine D) Amitriptyline

A ~ Fluvoxamine is indicated for the treatment of OCD and is classified as a selective serotonin reuptake inhibitor (SSRI). SSRIs are indicated for the treatment of depression, OCD, panic attacks, bulimia, premenstrual dysphoria disorder, posttraumatic stress disorder, social phobias, and social anxiety disorders. Phenelzine is indicated for depression not responsive to other agents. Desipramine and amitriptyline are tricyclic antidepressants indicated for treatment of depression especially if accompanied by anxiety or sleep disturbances.

The nurse is caring for a patient in a state of hypnosis, which means the patient is in what state? A) A state of extreme sedation in which the person no longer senses or reacts to incoming stimuli. B) A state of tranquility in which the person can be made to do whatever is suggested by others. C) A feeling of tension, nervousness, apprehension, or fear with high levels of awareness. D) A state in which the brain is no longer sending out signals to the body.

A ~ Hypnosis is an extreme state of sedation in which the person no longer senses or reacts to incoming stimuli. A state of tranquility is produced through minor tranquilizers by decreasing anxiety. Anxiety is a feeling of tension, nervousness, apprehension, or fear. Sedation is the loss of awareness and reaction to environmental stimuli, which may lead to drowsiness. The state of suggestibility often seen in television programs is not an appropriate definition of hypnosis. If the brain stopped sending signals, the patient would stop breathing and death would follow.

The client has been placed on Ativan. The nurse is planning a client instructional session. Which herbal preparation should the nurse emphasize that the client avoid taking with Ativan? a. Kava kava b. St. Johns wort c. Ginseng d. Ginger

A ~ Kava kava in combination with Ativan will increase the sedation effects of the Ativan.

A patient diagnosed with type 1 diabetes mellitus is receiving insulin. The physician has prescribed a monoamine oxidase inhibitor (MAOI) to treat this patients depression. What interaction will the nurse assess for with this drug combination? A) Increased risk of hypoglycemia B) Increased risk of hyperglycemia C) Increase in appetite D) Increased total cholesterol

A ~ MAOIs can cause an additive hypoglycemic effect if taken with insulin or oral diabetic agents. This patient would have to be monitored closely and appropriate dosage adjustments made; he should be taught the importance of more frequent blood sugar monitoring. The drug combination in this question would not cause an increase in appetite or increased total cholesterol.

The patient presents to the emergency department with a headache in the back of the head, palpitations, neck stiffness, nausea, vomiting, sweating, dilated pupils, tachycardia, and chest pain. Blood pressure measures 180/124 and heart rate is 168 beats per minute. The spouse says the only medication he takes is something for depression but she does not know the name of the drug and the patient is also unable to supply the name. What classification of antidepressant does the nurse suspect this patient is taking? A) Monoamine oxidase inhibitors (MAOIs) B) Selective serotonin reuptake inhibitors (SSRIs) C) Tricyclic antidepressants (TCAs) D) Antianxiety antidepressants

A ~ MAOIs have several serious adverse effects that can be fatal. This patients symptoms indicate fatal hypertensive crisis characterized by occipital headache, palpitations, neck stiffness, nausea, vomiting, sweating, dilated pupils, photophobia, tachycardia, and chest pain. It may progress to intracranial bleeding and fatal stroke. SSRIs and TCAs are not associated with these particular symptoms. Antianxiety antidepressants are not a classification of antidepressants.

The nurse is caring for a newborn who was delivered from a woman who took benzodiazepines for anxiety during the last 2 months of her pregnancy after her husband was killed in war. What will the nurse assess for in this newborn? A) Newborn withdrawal syndrome B) Hepatic dysfunction C) Failure to thrive D) Learning deficiencies

A ~ Neonatal withdrawal syndrome may result in a baby born to a mother who was taking benzodiazepines in the final weeks of pregnancy. The neonate may be given very small doses of benzodiazepines that are withdrawn gradually to prevent symptoms. Hepatic dysfunction in the neonate is not associated with use of benzodiazepines. Failure to thrive and learning deficiencies would be long-term problems and are not assessed during the neonatal period.

A patient has been taking Prozac (fluoxetine) for the past 3 years for depression. She is seeing her gynecologist for premenopausal symptoms and during the interview with the nurse she says that she would like to try Sarafem because her friend is taking it and she says it works great. The nurses best response is what? A) Sarafem and Prozac are different brand names for the same generic medication. B) Before changing drugs it is important to consider how well you responded to Prozac. C) You cannot take both drugs at the same time so it will be important to decide which is best. D) When taking both of these drugs, it is best to take one in the morning and one at night.

A ~ Prozac and Sarafem are different brand names for fluoxetine, so there is no benefit in changing the patients medication regimen and, if taken together, would result in a drug overdose. The other three responses are incorrect or inappropriate because they do not recognize that both drugs are the same.

The nurse is caring for an older adult in the long-term care facility who has begun to display signs of anxiety and insomnia. What is the priority nursing action? A) Assess the patient for physical problems. B) Call the provider and request an antianxiety drug order. C) Increase the patients social time, encouraging interaction with others. D) Suggest the family visit more often to reduce the residents stress level.

A ~ The patient should be screened for physical problems, neurological deterioration, or depression, which could contribute to the insomnia or anxiety. Only after physical problems are ruled out would the nurse consider nondrug measures such as increased socialization with other residents or family members. If nothing else is effective, pharmacological intervention may be necessary.

Which agent below is most likely to cause serious respiratory depression as a potential adverse reaction? A) Morphine (Duramorph) B) Pentazocine (Talwin) C) Hydrocodone (Lortab) D) Nalmefene (Revex)

A) Morphine (Duramorph) Morphine is a strong opioid agonist and as such has the highest likelihood of respiratory depression. Pentazocine, a partial agonist, and hydrocodone, a moderate to strong agonist, may cause respiratory depression but not as often and serious as morphine. Nalmefene is an opioid antagonist and would be used to reverse respiratory depression with opioids.

A postoperative patient has an epidural infusion of morphine sulfate (Astramorph). The patient's respiratory rate declines to 8 breaths/min. Which medication would the nurse anticipate administering? A) Naloxone (Narcan) B) Acetylcysteine (Mucomyst) C) Methylprednisolone (Solu-Medrol) D) Protamine sulfate

A) Naloxone (Narcan) Naloxone is a narcotic antagonist that can reverse the effects, both adverse and therapeutic, of opioid narcotic analgesics.

The nurse assesses the patient who had an abrupt withdrawal of benzodiazepines for withdrawal syndrome and would recognize what symptoms as part of the syndrome? (SATA) A) Headache B) Nightmares C) Malaise D) Bradycardia E) Hypotension

A, B, C ~ Abrupt cessation of benzodiazepines may lead to a withdrawal syndrome characterized by nausea, headache, vertigo, malaise, and nightmares. Withdrawal symptoms may be caused by the abrupt separation of benzodiazepine molecules from their receptor sites and the resulting acute decrease in gamma-aminobutyric acid (GABA) neurotransmission. Because GABA is an inhibitory neurotransmitter, less GABA may produce a less inhibited central nervous system (CNS) and therefore symptoms of hyperarousal or CNS stimulation. The nurse would not categorize hypotension or bradycardia as indicating benzodiazepine withdrawal.

The nurse is caring for a patient who received a new diagnosis of cancer. The patient exhibits signs of a sympathetic stress reaction. What signs and symptoms will the nurse assess in this patient consistent with an acute reaction to stress? (SATA) A) Profuse sweating B) Fast heart rate C) Rapid breathing D) Hypotension E) Inability to interact with others

A, B, C ~ Anxiety is often accompanied by signs and symptoms of the sympathetic stress reaction that may include sweating, fast heart rate, rapid breathing, and elevated blood pressure. Chronically anxious people may be afraid to interact with other people but this is not usually seen in an acute stress reaction.

The nurse interviews the family of a patient hospitalized with severe depression who is prescribed a tricyclic antidepressant. What assessment data are important in planning this patients plan of care? (SATA) A) Recent suicide attempts B) Gastrointestinal (GI) obstruction C) Affect D) Physical pain E) Personal responsibilities

A, B, C ~ When caring for a patient with a diagnosis of depression it is always important for the nurse to assess for recent suicide attempts, suicidal ideation, and any suicidal plans. After starting the medication, as the patient begins to feel better, risk of suicide increases, so ongoing assessment is essential to the patients safety. Other assessments include allergies, liver and kidney function, glaucoma, benign prostatic hypertrophy, cardiac dysfunction, GI obstruction, surgery, or recent myocardial infarction, all of which could be exacerbated by the effects of the drug. Assess history of psychiatric problems, or myelography within the past 24 hours or in the next 48 hours, or is taking a monoamine oxidase inhibitor to avoid potentially serious adverse reactions. Physical pain and personal responsibilities may be assessed but are not priority assessments unless indicated by other diagnoses.

The nurse is teaching a class for nurses working in prenatal clinics about the danger associated with use of benzodiazepines during pregnancy and explains that what fetal anomalies result from maternal use of benzodiazepines during the first trimester of pregnancy? (SATA) A) Cleft lip or palate B) Inguinal hernia C) Cardiac defects D) Microencephaly E) Gastroschises

A, B, C, D ~ Benzodiazepines are contraindicated in pregnancy because a predictable syndrome of cleft lip or palate, inguinal hernia, cardiac defects, microcephaly, or pyloric stenosis occurs when they are taken in the first trimester. Gastroschises, when the abdominal organs are found outside the abdominal cavity, is not associated with use of benzodiazepine use in the first trimester.

What reasons can the nurse give for why barbiturates are no longer considered the mainstay for treatment of anxiety? (SATA) A) Adverse effects are more severe. B) There is an increased risk of physical tolerance. C) There is an increased risk of psychological dependence. D) The most common adverse effects are related to cardiac arrhythmias. E) Hypersensitivity reactions can sometimes be fatal.

A, B, C, E ~ The adverse effects caused by barbiturates are more severe than those associated with other, newer sedatives/hypnotics. For this reason, barbiturates are no longer considered the mainstay for the treatment of anxiety. In addition, the development of physical tolerance and psychological dependence is more likely with the barbiturates than with other anxiolytics. The most common adverse effects are related to central nervous system (CNS) depression. Hypersensitivity reactions to barbiturates are sometimes fatal.

The nurse is teaching a patient about foods to avoid when taking isocarboxazid (Marplan). Which foods will the nurse instruct the patient to avoid? (Select all that apply.) a. Bananas b. Bread c. Eggs d. Red wine e. Sausage f. Yogurt

A, D, E, F

Which are characteristics sons of information? A. Edema B. Erythema C. Heat D. Numbness E. Pallor F. Paresthesia

A,B,C

Which patient may require a higher than expected dose of opioid analgesic A. A patient with cancer B. A patient with a concussion C. A patient with hypotension D. A patient three days after surgery

A. A patient with cancer

The parent of a five-year-old child ask the nurse to recommend an over-the-counter pain medication for the child. Which Analgesic will the nurse recommend A. Acetaminophen (Tylenol) B. Aspirin (Ecotrin) C. Diflunisal (Dolobid) D. Ibuprofen (Motrin)

A. Acetaminophen (Tylenol)

The nurse is caring for a patient who is in mid it with a fractured leg in for observation of a closed head injury after a motor vehicle accident. The patient reports having pain at a level of three on a 1 to 10 pain scale. The nurse will expect the provider to order which analgesic medication for this patient A. Acetaminophen (Tylenol) PO B. hydromorhone HCl (Dilaudid) IM C. Morphine sulfate PCA D. Transdermal fentanyl (Duragesic)

A. Acetaminophen (Tylenol) PO

Which antigout medication is used to treat chronic tophaceous gout? A. Allopurinol (Zyloprim) B. Colchicine C. Probenecid (Benemid) D. Sulfinpyrazone (Anturane)

A. Allopurinol (Zyloprim)

The nurse is performing a health history on a patient who has arthritis. The patient reports tinnitus. Suspecting a drug adverse affect, the nurse will ask the patient about which medication? A. Aspirin (Bayer) B. Acetaminophen (Tylenol) C. Anakinra (Kineret) D. Prednisone (Deltasone)

A. Aspirin (Bayer)

A patient who takes high dose aspirin to treat rheumatoid arthritis has a serum's salicylate level of 35 mg/dL her. The nurse will perform which action? A. Assess the patient for tinnitus B. Monitor the patient for signs of Reye's syndrome C. Notify the provider of severe aspirin toxicity D. Request an order for an increased aspirin does

A. Assess the patient for tinnitus

A post operative patient has a history of opioid abuse. Which analgesic medication will the nurse expect the provider to order for this patient A. Buprenorphine (Buprenex) B. Butorphanol tartrate (Stadol) C. Naloxone (Narcan) D. Pentazocine (Talwin)

A. Buprenorphine (Buprenex

The nurse is caring for a post operative older patient who received PO hydrocodone with acetaminophen (Lortab) 45 minutes prior after reporting a pain level of eight on a scale of 1 to 10. The patient reports a pain level of four, and the nurses notes A respiratory rate of 20 breaths per minute, a heart rate of 92 bpm, and a blood pressure of 170/95 mm Hg. Which action will the nurse take? A. Contact the provider and request an order for more potent opioid analgesic B. Reassess the patient in 30 minutes C. Request an order for ibuprofen to augment the opioid Analgesic D. Suggest that the patient use nonpharmacological measures to relieve pain

A. Contact the provider and request an order for more potent opioid analgesic

A nursing student asked the nurse to explain the role of cyclooxygenase-2 (COX-2) and its role in information. The nurse will explain that COX-2 a. Converts arachidonic acid into a chemical mediator for inflammation b. Directly causes vasodilation and increased capillary permeability c. Irritates the gastric mucosa to cause gastrointestinal upset d. Releases prostaglandins, which cause inflammation and pain and tissues

A. Converts arachidonic acid into a chemical mediator for inflammation

A patient is taking ibuprofen 400 mg every four hours to treat moderate arthritis pain and reports that it is less effective than before. What action will the nurse take A. Counseled the patient to discuss the prescription NSAID with the provider B. Recommend adding aspirin to increase the anti-inflammatory effect C. Suggest asking the provider about a short course of corticosteroids D. Tell the patient to increase the dose to 800 mg every four hours

A. Counseled the patient to discuss the prescription NSAID with the provider

The nurse is assessing a patient who has gal who will begin taking Allopurinol (Zyloprim). The nurse reviews the patient's medical record and will be concerned about which laboratory results? A. Elevated BUN and creatinine B. Increased serum uric acid C. Slight increase in the white blood count D. Increased serum glucose

A. Elevated BUN and creatinine

The nurse is teaching a female patient who will begin taking two tablets of 325 mg acetaminophen every 4 to 6 hours as needed for pain. Which statement by the patient indicates understanding of the teaching A. I may take acetaminophen up to six times daily if needed B. I should increase the dose of acetaminophen if I drink caffeinated coffee C. If I take oral contraceptive pills, I should be back up contraception D. It is safe to take acetaminophen with any over-the-counter medications

A. I may take acetaminophen up to six times daily if needed

The nurse is providing teaching to a patient who will begin taking aspirin to treat arthritis pain. Which statement by the patient indicates a need for further teaching A. I should increase fiber and fluids while taking aspirin B. I will call my provider if I have abdominal pain C. I will drink a full glass of water with each dose D. I will notify the provider of ringing in my ears

A. I should increase fiber and fluids while taking aspirin

A patient who is seven months pregnant and who has arthritis asked the nurse if she can take aspirin for pain. The nurse will tell her not to take aspirin for which reason? A. It can result in adverse effects on her fetus B. It causes an increase of Reye's syndrome C. It increases hemorrhage risk D. It will cause increased gastrointestinal distress

A. It can result in adverse effects on her fetus

A patient who has osteoarthritis with mild to moderate pain asked the nurse about taking over-the-counter ibuprofen (Motrin). What will the nurse tell this patient? A. It may take several weeks to achieve therapeutic effects B. Unlike aspirin there is no increased risk of bleeding with ibuprofen C. Take ibuprofen twice daily for maximum analgesic benefit D. Combine ibuprofen with acetaminophen for best effect

A. It may take several weeks to achieve therapeutic effects

Uric Acid Inhibitor (Allopurinol/anti-gout drugs)

Action: decreases uric acid synthesis, prevents gout attacks Side effects: · Dizziness, headache, dry mouth, GI distress · Arthralgia, pruritis, erectile dysfunction · Bradycardia, hyperglycemia · Blood dyscrasias, Steven-Johnson

Colchicine (anti-gout drug)

Action: inhibits migration of leukocytes to inflamed site, alleviates gout symptoms Side effects: GI distress, take with food Contraindication: severe renal, cardiac, or GI problems

Morphine

Actions: · Effective against pain from MI and cancer · Relieves symptoms from pulmonary edema · Pre-op med, relieves anxiety Side effects/adverse: · Drowsiness, dizziness, euphoria · Confusion, depression, miosis, blurred vision, · Resp depression Morphine is the opioid analgesic prototype - all others measured in comparison Pt care interventions: · Administer before pain reaches peak · Monitor vitals · Check for pupil changes · Have naloxone available as antidote

A nurse is caring for a cancer patient receiving subcutaneous morphine sulfate for pain. Which of the following nursing actions is most important in the care of this patient? A. Monitor urine output. B. Monitor respiratory rate. C. Monitor heart rate. D. Monitor temperature.

Answer: B Morphine sulfate can suppress respiration and respiratory reflexes, such as cough. Patients should be monitored regularly for these effects to avoid respiratory compromise. Morphine sulfate does not significantly affect urine output, heart rate, or body temperature.

A patient arrives at the emergency department complaining of back pain. He reports taking at least 3 acetaminophen tablets every three hours for the past week without relief. Which of the following symptoms suggests acetaminophen toxicity? A. Tinnitus. B. Diarrhea. C. Hypertension. D. Hepatic damage.

Answer: D Acetaminophen in even modestly large doses can cause serious liver damage that may result in death. Immediate evaluation of liver function is indicated with consideration of N-acetylcysteine administration as an antidote. Tinnitus is associated with ASPIRIN overdose, not acetaminophen. Diarrhea and hypertension are not associated with acetaminophen.

The mental health nursing instructor is talking with the class about depression. What deficiency does the instructor explain will result in depression? A) Epinephrine, norepinephrine, and acetylcholine B) Norepinephrine, dopamine, and serotonin C) Acetylcholine, gamma-aminobutyric acid, and serotonin D) Gamma-aminobutyric acid, dopamine, and epinephrine

B ~ A current hypothesis regarding the cause of depression is a deficiency of norepinephrine, dopamine, or serotonin, which are all biogenic amines, in key areas of the brain. Acetylcholine is a neurotransmitter that communicates between nerves and muscles. Epinephrine is a catecholamine that serves as a neurotransmitter that is released in the sympathetic branch of the autonomic nervous system and can be hormones when released from cells in the adrenal medulla. Gamma-aminobutyric acid is a neurotransmitter that inhibits nerve activity and prevents over excitability or stimulation.

A client is experiencing severe EPS effects. In addition to administering a lower dose of the antipsychotic agents, the nurse would anticipate administering a medication in which category? a. Cholinergics b. Anticholinergics c. Antidepressants d. Dopamine agonists

B ~ Anticholinergics, such as benztropine (Cogentin), are used to decrease the EPS effects associated with antipsychotic medications.

Client teaching is important when antipsychotics are taken after discharge from the hospital. Nursing instruction should include giving which information to the client and family? a. Therapeutic effect should occur in 2 to 3 days with maximum effect in 1 week. b. The drugs should not be discontinued without consulting a healthcare provider. c. Taking barbiturates in small dosages with the drug is usually permissible. d. Rapid change in position has little effect on dizziness or blood pressure.

B ~ Antipsychotic medications affect symptoms while they are used. If they are stopped, the symptoms will recur.

A client is brought to the emergency department unconscious. The clients spouse tells the nurse that the client was found in bed with an empty pill bottle nearby. The clients spouse believes that there were 20 to 25 diazepam (Valium) pills in the bottle. What represents an appropriate nursing priority? a. Administer an emetic agent followed by activated charcoal. b. Lavage the stomach using a nasogastric tube. c. Prepare the client for emergency surgery. d. Monitor the client because there is no antidote.

B ~ Because the client is unconscious, this is the correct course of action.

The nurse is caring for a patient who has a sedative hypnotic ordered. The nurse would consider this drug contraindicated if the patient had what disorder? A) Neurological diseases B) Liver failure C) Endocrine disorders D) Heart disease

B ~ Benzodiazepines undergo extensive hepatic metabolism. In the presence of liver disease, the metabolism of most benzodiazepines is slowed, with resultant accumulation and increased risk of adverse effects. Neurological disorders, endocrine disorders, and heart disease are not contraindications for the use of benzodiazepines.

What anxiolytic drugs would be given to a premenopausal patient who is a registered nurse planning to return to work at the hospital after anxiety is controlled? A) Alprazolam (Xanax) B) Buspirone (BuSpar) C) Diazepam (Valium) D) Clorazepate (Tranxene)

B ~ Buspirone is a newer anxiolytic drug that does not cause sedation or muscle relaxation. It is preferred when the patient needs to be alert such as when driving or working. Alprazolam, diazepam, and clorazepate are benzodiazepines, which cause drowsiness, sedation, depression, lethargy, confusion, and decreased mental alertness. It would be unsafe for a nurse to function in her role while taking one of these drugs.

When compared with benzodiazepines, buspirone (BuSpar) stands out as unique among antianxiety drugs because of what factor? A) Increases the central nervous system (CNS) depression of alcohol and other drugs. B) Lacks muscle relaxant and anticonvulsant effects. C) Causes significant physical and psychological dependence. D) Rapidly absorbed from the gastrointestinal (GI) tract and metabolized in the liver.

B ~ Buspirone, a newer antianxiety agent, has no sedative, anticonvulsant, or muscle-relaxant properties, and its mechanism of action is unknown. However, it reduces the signs and symptoms of anxiety without many of the central nervous system effects and severe adverse effects associated with other anxiolytic drugs. Most of the antianxiety drugs are rapidly absorbed from the GI tract, metabolized in the liver, have a significant drugdrug interaction with alcohol and other drugs, and can result in psychological dependence.

The patient has been severely depressed since her father died 6 months ago. The physician has prescribed amitriptyline. The nurse reviews the patients chart before administering the medication. What preexisting condition would require cautious use of this drug? A) Osteosarcoma B) Cardiovascular disorders C) Closed head injury D) Bleeding ulcer

B ~ Caution should be used with tricyclic antidepressants in patients with preexisting cardiovascular (CV) disorders because of the cardiac stimulatory effects of the drug and with any condition that would be exacerbated by the anticholinergic effects (e.g., angle-closure glaucoma, urinary retention, prostate hypertrophy, GI or genitourinary surgery). There is no indication that caution is needed with patients diagnosed with osteosarcoma, closed head injury, or bleeding ulcer.

The client has been placed on Risperdal. He complains to the nurse of experiencing headaches. The highest priority action on the part of the nurse is to recognize that this is a(n) ________ the medication and call the physician. a. adverse reaction to b. expected side effect of c. life-threatening reaction to d. anaphylactic reaction to

B ~ Headaches are an expected side effect of treatment with Risperdal.

An elderly patient has been taking zolpidem (Ambien) as a sleep aid for the past 2 months. On admission to the assisted-living facility, it is determined that the drug is no longer needed. What is an important nursing consideration concerning this drug? A) Hallucinations are common. B) The drug needs to be withdrawn gradually. C) Another anxiolytic will need to be substituted. D) Sundowning is common with withdrawal from this drug.

B ~ It is important for the nurse to understand that zolpidem must be withdrawn gradually over a 2-week period after prolonged use. If chloral hydrate is stopped suddenly, it will result in serious adverse effects. Hallucinations and sundowning are not common with withdrawal of the drug. The prescriber and the patient would determine the need for chloral hydrate to be substituted for another anxiolytic.

The client is known to have overdosed on a benzodiazepine medication. The nurse anticipates that which medication will most likely be ordered? a. Tranxene b. Romazicon c. BuSpar d. Librium

B ~ Romazicon is considered to be the benzodiazepine antagonist.

The nurse is caring for a resident in a long-term care facility who is African American with a history of an anxiety disorder. The patient is receiving oral lorazepam (Ativan) 2 mg t.i.d. When developing this patients plan of care, what priority assessment will the nurse include? A) Depression B) Extreme sedation C) Phlebitis D) Nightmares

B ~ Special care should be taken when anxiolytic or hypnotic drugs are given to African Americans. About 15% to 20% of African Americans are genetically predisposed to delayed metabolism of benzodiazepines. As a result, they may develop high serum levels of these drugs, with increased sedation and an increased incidence of adverse effects. Depression is not a common adverse effect. Phlebitis can occur at injection sites but this patient is taking the medication orally. Nightmares occur during drug withdrawal.

Why would the nurse expect the patient with liver disease to receive a smaller dose of benzodiazepines? A) Excretion of the drug relies on liver function. B) The drugs are metabolized extensively in the liver. C) They are lipid soluble and well distributed throughout the body. D) The drugs are well absorbed from the gastrointestinal tract.

B ~ The benzodiazepines are metabolized extensively in the liver. Patients with liver disease must receive a smaller dose and be monitored closely. Excretion is primarily through the urine. All of the answer options are true, but only the fact that the benzodiazepines are metabolized in the liver explains why a patient with liver disease would require smaller dosages.

The nurse is caring for a patient in intensive care unit receiving IV lorazepam (Ativan) to reduce anxiety related to mechanical ventilation. While injecting the medication the nurse notes a decrease in blood pressure and bradycardia. What is the nurses priority action? A) Discontinue drug administration. B) Give the IV drug more slowly. C) Notify the patients health care provider. D) Document the reaction to the drug.

B ~ The nurses priority action is to slow the rate of injection because rapid injection of benzodiazepines can result in hypotension and bradycardia and can lead to cardiac arrest.

A nurse is discussing the use of alprazolam (Xanax) with a 68-year-old patient. What statement indicates that the patient has an understanding of the drug? A) When I stop having panic attacks, I can stop taking the drug. B) This drug will calm me down in about 30 minutes after I take it. C) One dose will keep me calm for about 24 hours. D) I am taking an increased dose because of my age.

B ~ The onset of alprazolam is about 30 minutes. The drug must be tapered after long-term use and the duration is approximately 4 to 6 hours. Elderly patients usually have a reduced dosage.

The nurse evaluates teaching as effective when a patient taking a benzodiazepine states, A) I should always take the medication with meals. B) I should not stop taking this drug without talking to my health care provider first. C) I cannot take aspirin with this medication. D) I will have to take this medication for the rest of my life.

B ~ The patient makes a correct statement when saying the drug should not be stopped without talking to the health care provider first because withdrawal of benzodiazepines require careful monitoring and should be gradually withdrawn. Medications do not have to be taken with food, aspirin is not contraindicated, and the medication need only be taken while the condition being treated continues. Patients with anxiety may only need the medication for a few weeks whereas those with a seizure disorder may take it for longer periods of time.

A client is to receive a dose of fluphenazine hydrochloride (Prolixin) by intramuscular injection. What is the most important nursing intervention related to the injection? a. Massage the site vigorously after injection. b. Administer the drug using Z-tracking. c. Avoid rotating the injection sites. d. Select a 22- to 23-gauge needle.

B ~ This medication is very viscous and requires Z-track, deep IM injection to avoid muscle irritation.

The nurse is caring for a patient who is taking a benzodiazepine. The nurse knows that caution should be used when administering a benzodiazepine to the elderly because of what possible adverse effect? A) Acute renal failure B) Unpredictable reactions C) Paranoia D) Hallucinations

B ~ Use benzodiazepines with caution in elderly or debilitated patients because of the possibility of unpredictable reactions and in patients with renal or hepatic dysfunction, which may alter the metabolism and excretion of these drugs, resulting in direct toxicity. Dosage adjustments usually are needed for such patients. Acute renal failure, paranoia, and hallucinations are not commonly related to therapy with these medications in the elderly.

What drug, if prescribed for the patient, would indicate the need to assess the patient for depression characterized by anxiety and addictive behaviors? A) Imipramine (Tofranil) B) Venlafaxine (Effexor) C) Fluvoxamine (Luvox) D) Tranylcypromine (Parnate)

B ~ Venlafaxine is used to treat and prevent depression in generalized anxiety disorder, social anxiety disorder; it also diminishes addictive behavior. Fluvoxamine is a selective serotonin reuptake inhibitor, tranylcypromine is a monoamine oxidase inhibitor, and imipramine is a tricyclic antidepressant that are not indicated for treatment of anxiety disorder and addictive behavior.

A patient admitted to the hospital with a diagnosis of pneumonia asks the nurse, "Why am I receiving codeine? I don't have any pain." The nurse's response is based on the knowledge that codeine also has which effect? A) Immunostimulant B) Antitussive C) Expectorant D) Immunosuppressant

B) Antitussive Codeine provides both analgesic and antitussive therapeutic effects.

The nurse is planning care for a patient receiving morphine sulfate (Duramorph) by means of a patient-controlled analgesia (PCA) pump. Which intervention may be required because of a potential adverse effect of this drug? A) Administer cough suppressant. B) Insert Foley catheter. C) Administer antidiarrheal. D) Monitor liver function tests.

B) Insert Foley catheter. Morphine can cause urinary hesitancy and urinary retention. If bladder distention or the inability to void is noted, the prescriber should be notified. Urinary catheterization may be required. Morphine acts as a cough suppressant and an antidiarrheal, so neither of those drugs would need to be administered to counteract an adverse effect of morphine. Liver toxicity is not a common adverse effect of morphine.

The nurse is working on a postoperative unit where pain management is part of routine care. Which statement below is the most helpful in guiding clinical practice in this setting? A) At least 30% of the U.S. population is prone to drug addiction and abuse. B) The development of opioid dependence is rare when opioids are used for acute pain. C) Morphine is a common drug of abuse in the general population. D) The use of PRN (as needed) dosing provides the most consistent pain relief without risk of addiction.

B) The development of opioid dependence is rare when opioids are used for acute pain.

A patient with severe depression has been hospitalized and the physician has ordered amitriptyline. What common adverse effect will the nurse monitor and assess the patient for? (SATA) A) Fever B) Myocardial Infarction C) Stroke D) Dry mouth E) Gynecomastia

B, C, D ~ Use of tricyclic antidepressants may lead to GI anticholinergic effects, such as dry mouth, constipation, nausea, vomiting, anorexia, increased salivation, cramps, and diarrhea. Cardiovascular effects (e.g., orthostatic hypotension, hypertension, arrhythmias, myocardial infarction, angina, palpitations, stroke) may occur. Fever and gynecomastia are not normally attributed to amitriptyline therapy.

What priority teaching point does the nurse include in the teaching plan for a patient on a monoamine oxidase inhibitor (MAOI)? (SATA) A) Take medication at bedtime. B) Monitor blood pressure. C) Do not take over-the-counter (OTC) drugs without talking to physician. D) Report double vision right away E) Reduce tyramine intake

B, C, E ~ MAOIs can cause drug-drug and drug-food interactions, which can precipitate cardiovascular effects that include orthostatic hypotension, arrhythmias, palpitations, angina, and the potentially fatal hypertensive crisis. Priority teaching points include monitoring blood pressure which will elevate with tyramine ingestion and the importance of not taking any OTC without physician or pharmacist consultation due to multiple drug-drug interactions. When taking an MAOI, you would not necessarily take the drug at bedtime or drink lots of fluid. Blurred, but not double, vision is an adverse effect of an MAOI.

The nurse is teaching a patient about using high dose aspirin to treat arthritis. What information will the nurse include when teaching this patient? A. A normal serum Aspirin level is between 30 and 40 mg/dL B. "You may need to start taking this drug a week prior to surgery" C. You will need to monitor aspirin levels if you are also taking warfarin D. Your stores may become dark, but this is a Harmless side effect

B. "You may need to stop taking this drug a week prior to surgery"

The nurse is preforming an admission assessment on a stable patient admitted after a motor vehicle accident. The patient reports having "bad pain." What will the nurse do first A. Administer acetaminophen (Tylenol) B. Ask the patient to rate the pain on a 1-10 scale. C. Attempt to determine what type of pain the patient has D. Request an order for an intravenous opioid Analgesic

B. Ask the patient to rate the pain on a 1-10 scale

The nurses caring for a patient who has rheumatoid arthritis and who is receiving infliximab (Remicade) IV every eight weeks. Which the laboratory test will this nurse anticipate that this patient will need? A. Calcium level B. Complete blood count C. Electrolytes D. Potassium

B. Complete blood count

The nurse assesses an older patient 60 minutes after administering 4 mg of intravenous morphine sulfate (MS) for postoperative pain. The patience analgesia order is for 2 to 5 mg of MS IV every two hours. The nurse notes that the patient is lying very still. The patient's heart rate is 96 bpm, respiratory rate is 14 breaths per minute, and blood pressure is 140/90 mm Hg. When asked to rate the level of pain, The patient replies "just a five". The nurse will perform which action. A. Give 3 mg of MS at the next dose B. Give 5 mg of MS at the next dose C. Request an order for an oral opioid to give now D. Request an order for acetaminophen to give now

B. Give 5 mg of MS at the next dose

The nurse provides teaching for a patient who will begin taking idomethacin (Inderal) to treat rheumatoid arthritis. Which David by the patient indicates a need for further teaching? A. I should limit sodium intake while taking this drug B. I should take Idomethacin on an empty stomach C. I will need to check my blood pressure frequently D. I will take the medication twice daily

B. I should take idomethacin on an empty stomach

The nurse provides teaching for a patient who will begin taking allopurinol. Which statement by the patient indicates understanding of the teaching A. I should increase my vitamin C intake B. I will get yearly eye exams C. I will increase my protein intake D. I will limit fluids to prevent edema

B. I will get yearly eye exams

A nursing student asks how nonsteroidal anti-inflammatory drugs (NSAIDs) work to suppress inflammation and reduce pain. The nurse will explain the NSAIDs a. Exert direct actions to cause relaxation of smooth muscle b. Inhibit cyclooxygenase that is necessary for prostaglandin synthesis c. Interfere with neuronal pathways associated with prostaglandin action d. Suppress prostaglandin activity by blocking tissue receptor sites

B. Inhibit cyclooxygenase that is necessary for prostaglandin synthesis

An adolescent female has dysmenorrhea associated with heavy menstrual periods. The patient provider has recommended ibuprofen (Motrin). When teaching this patient about this drug the nurse will tell her that ibuprofen A. May decrease the effectiveness of oral contraceptive pills B. May increase bleeding during her period C. Should be taken on an empty stomach to increase absorption D. Will decrease the duration of her periods

B. May increase bleeding during her period

The nurse checks on a patient who has received sumatriptan (Imitrex) for treatment of a migraine headache. The patient reports moderate improvement in headache pain and reports feeling dizzy. The nurse notes a blood pressure of 160/85 mm Hg. Which action by the nurse is correct A. Notify the provider of the dizziness B. Notify the provider of the increased blood pressure C. Plan to administer a second dose in one hour D. Request an order for intranasal sumatriptan

B. Notify the provider of the increased blood pressure

The nurse is caring for a patient who has been taking an NSAID for four weeks for osteoarthritis. The patient reports decreased pain, but the nurse notes continued swelling of the affected joints. The nurse will perform which action A. Assess the patient for drug seeking behaviors B. Notify the provider the drug is not effective C. Reassure the patient that swelling will decrease eventually D. Remind the patient that this drug is given for pain only

B. Notify the provider the drug is not effective

The nurse assumes care of a patient in the post anesthesia care unit. The patient had abdominal surgery and is receiving intravenous morphine sulfate for pain. The patient is asleep and has not voided since prior to the surgery. The nurse assesses the respiratory rate of 10 breaths per minute and Notes hypoactive bowel sounds. The nurse will contact the surgeon to report which condition A. Paralytic ileus B. Respiratory depression C. Somnolence D. Urinary retention

B. Respiratory depression

The nurse is discussing celecoxib (Celebrex) with a patient who use the drug to treat dysmenorrhea. What information will the nurse include in teaching? A. Do not take the medication during the first two days of your period. B. The initial dose will be twice the amount of subsequent doses C. Take this medication with food to minimize gastrointestinal upset D. Take the draws on a regular basis to prevent dysmenorrhea

B. The initial dose will be twice the amount of subsequent doses

What would the nurse assess for when benzodiazepines are abruptly stopped? A) Urinary retention and change in sexual functioning B) Dry mouth, constipation, nausea, and vomiting C) Nausea, headache, vertigo, malaise, and nightmares D) In most cases nothing significant

C ~ Abrupt cessation of these drugs may lead to a withdrawal syndrome characterized by nausea, headache, vertigo, malaise, and nightmares. When benzodiazepines are stopped abruptly the likelihood of withdrawal symptoms increases with the length of time the patient took the medication. Urinary retention, change in sexual functioning, dry mouth, constipation, nausea, and vomiting are all common adverse effects of the medications classified as benzodiazepines.

A patient presents at the free clinic complaining of nervousness, worrying about everything, and feeling very tense. What diagnose would the nurse suspect? A) Neurosis B) Psychosis C) Anxiety D) Depression

C ~ Anxiety is a common disorder that may be referred to as nervousness, tension, worry, or using other terms that denote an unpleasant feeling. The other options would not be described by these symptoms.

A patient arrives at the emergency room after attempting suicide by taking an entire bottle of diazepam. What antidote will the nurse most likely administer? A) Phenobarbital (Luminal) B) Dexmedetomidine (Precedex) C) Flumazenil (Romazicon) D) Ramelteon (Rozerem)

C ~ Flumazenil is an antidote to benzodiazepine overdose and is administered to reverse the effects of benzodiazepines when used for anesthesia. Phenobarbital, a barbiturate, would further depress the body functions of this patient. Dexmedetomidine is a new hypnotic drug used in the intensive care unit for mechanically ventilated patients. Ramelteon is also new; it is used as a hypnotic. Adverse effects of this drug include depression and suicidal ideation.

A nurse is working with a 9-year-old child who exhibits signs and symptoms of obsessive-compulsive disorder (OCD). What drug will the nurse anticipate may be prescribed for the child? A) Phenelzine (Nardil) B) Amitriptyline (Elavil) C) Fluvoxamine (Prozac) D) Isocarboxazid (Marplan)

C ~ Fluvoxamine is a selective serotonin reuptake inhibitor that has established pediatric dosage guidelines for the treatment of obsessive-compulsive disorder. Isocarboxazid and phenelzine are monoamine oxidase inhibitors and should be avoided in pediatric use because of the potential drugfood interactions and other serious adverse effects. Amitriptyline is also a tricyclic antidepressant not recommended for pediatric use.

Hypnotic drugs are used to aid people in falling asleep. What physiological system does a hypnotic act on to be effective in helping a patient to sleep? A) Limbic system B) Sympathetic nervous system C) Reticular activating system D) Lymph system

C ~ Hypnotics are used to help people fall asleep by causing sedation. Drugs that are effective hypnotics act on the reticular activating system and block the brains response to incoming stimuli. Hypnosis, therefore, is the extreme state of sedation, in which the person no longer senses or reacts to incoming stimuli. The other options are incorrect.

The client has been diagnosed with schizophrenia and is exhibiting a loss of function and motivation. The nurse recognizes that these symptoms are categorized as: a. positive. b. paranoiac. c. negative. d. incoherent.

C ~ Negative symptoms are manifested as the inability to initiate voluntary motor function. The others are considered positive symptoms.

The nurse is caring for a 36-year-old man who experienced a seizure 30 minutes before coming into the emergency room, where he begins to have another. What barbiturate has the fastest onset and would be most appropriate to give to the patient to quickly stop the seizure? A) Amobarbital (Amytal Sodium) B) Mephobarbital (Mebaral) C) Phenobarbital (Luminal) D) Secobarbital (Seconal)

C ~ Phenobarbitals onset is between 10 and 60 minutes, depending on the route administered, and most likely this would be given to the patient. Amobarbital is given for convulsions and the onset is between 15 and 60 minutes. Mephobarbitals onset is between 30 and 60 minutes. Secobarbital is given for convulsive seizures of tetanus and has an onset of 1 to 4 hours.

A nurse is caring for a 4-year-old child who is receiving a barbiturate. What common adverse effect would the nurse assess for? A) Decrease in respirations B) Vomiting C) Excitability D) Dry mucous membranes

C ~ The barbiturates, being older drugs, have established pediatric dosages. These drugs must be used with caution because of the often unexpected responses. Children must be monitored very closely for central nervous system (CNS) depression and excitability. The most common adverse effects are related to general CNS depression. Other CNS effects may include drowsiness, somnolence, lethargy, ataxia, vertigo, a feeling of a hangover, thinking abnormalities, paradoxical excitement, anxiety, and hallucinations. Alteration in respirations and dried mucous membranes are adverse effects of antihistamines, which can be given to calm children or induce sleep. Vomiting could occur with the use of paraldehyde due to the unpleasant taste and odor of the drug.

A client has been taking a phenothiazine for 1 week. She contacts the crisis intervention clinic because she is still having symptoms. The nurses response is based on the premise that the desired effects usually take _____ to manifest. a. 1 week b. 1 to 3 weeks c. 3 to 6 weeks d. 3 to 5 months

C ~ The client may feel some effect in 7 to 10 days, but generally it takes 6 weeks for the medication to take full effect.

The client is an older adult who has been placed on Librium. The nurse recognizes that the dose of the drug _____ for this client. a. is contraindicated b. should be increased c. should be decreased d. will not change

C ~ The dose of Librium should be decreased for an older adult.

The nurse is caring for a patient who experiences anxiety and insomnia and is prescribed benzodiazepines. When developing the plan of care, what would be an appropriate nursing diagnosis related to potential adverse effects of the drug? A) Provide patient teaching about drug therapy. B) Anxiety related to drug therapy. C) Risk for injury related to central nervous system (CNS) effects. D) Avoid preventable adverse effects, including abuse and dependence.

C ~ The most appropriate nursing diagnosis related to adverse effects of the drug is risk for injury related to CNS effects because benzodiazepines can have many CNS adverse effects. Anxiety is the condition for which drug therapy is prescribed not related to drug therapy. Patient teaching and avoiding adverse effects are interventions and not nursing diagnoses.

A client is to be treated with Prolixin. The highest priority nursing intervention related to the clients vital signs is to monitor for: a. bradycardia. b. hypertension. c. hypotension. d. tachypnea.

C ~ The phenothiazine groups major side effect is hypotension.

A nurse is about to administer a parenteral benzodiazepine to a female patient in the hospital before the performance of a procedure. What is the priority nursing action before administration of the drug? A) Make sure that the side rails are up and the bed is in the lowest position. B) Close the blinds and ensure appropriate room temperature for the patient. C) Help the patient out of bed to the bathroom and encourage her to void. D) Ask all visitors to leave the room and remain in the waiting area.

C ~ The priority action would be to help the patient up to void. After the medication is administered the patient should not get out of bed because of possibly injury due to drowsiness. Safety should always be the priority concern. After administration of the drug the nurse would ask visitors to leave before beginning the procedure, make the room conducive to rest and sleep, and make sure that both side rails are up and the bed is in the lowest position.

A client is ordered to receive fluphenazine (Prolixin) to manage the psychotic symptoms of schizophrenia. The nurse assesses for which signs of anticholinergic effects? a. Bradycardia and orthostatic hypotension b. Diarrhea and tachycardia c. Urinary retention and dry mouth d. Constipation and hypertension

C ~ Urinary retention & dry mouth are the side effects of anticholinergics.

The healthcare provider prescribes naproxen (Naproxen) twice daily for a client with osteoarthritis of the hands. The client tells the nurse that the drug does not seem to be effective after three weeks. Which is the best response for the nurse to provide? A) The frequency of the dosing is necessary to increase the effectiveness. B) Therapeutic blood levels of this drug are reached in 4 to 6 weeks. C) Another type of nonsteroidal antiinflammatory drug may be indicated. D) Systemic corticosteroids are the next drugs of choice for pain relief

C) Another type of nonsteroidal antiinflammatory drug may be indicated.

A nurse administers naloxone (Narcan) to a postoperative patient experiencing respiratory sedation. What undesirable effect would the nurse anticipate after giving this medication? A) Drowsiness B) Tics and tremors C) Increased pain D) Nausea and vomiting

C) Increased pain Naloxone is a medication that reverses the effects of narcotics. Although the patient's respiratory status will improve after the administration of naloxone, pain will be more acute.

The patient is taking aspirin to help prevent myocardial infarction and is experiencing moderate gastrointestinal upset. The nurse will contact the patients provider to discuss changing from aspirin to which drug A. A Cox-2 inhibitor B. celecoxib (Celebrex) C. Enteric coated aspirin D. Nabumetone (Relafen)

C. Enteric-coated aspirin

The nurse is caring for a six-year-old child who had surgery that morning. The child is awake and laying very still in bed and won't respond when the nurse asked about pain. The nurse to perform which action A. Ask the child to rate the pain on a scale of 1 to 10 B. Encourage the child to request pain medication when needed C. Evaluate the child's pain using an "ouch" scale D. Plan to administer pain medication if the child begins to cry

C. Evaluate the child's pain using an "ouch" scale

The nurse is teaching a patient about taking colchicine to treat gout. What information will the nurse include when teaching this patient about this drug? A. Avoid all alcohol except beer B. Include salmon in the diet C. Increase fluid intake D. Take on an empty stomach

C. Increase fluid intake

A patient who is taking aspirin for arthritis pain asked the nurse why also causes gastrointestinal upset. The nurse understands that this is because aspirin A. Increases gastrointestinal secretions B. Increases hypersensitivity reactions C. Inhibits both COX-1 and COX-2 D. Is an acidic compound

C. Inhibits both COX-1 and COX-2

The nurse is caring for a postpartum woman who is refusing opioid analgesics but is reading her pain is a seven or eight on a 10 point scale. The nurse will contact the provider to request an order for which Analgesic medication? A. Diclofenac sodium (Voltaren) B. Ketoprofen (Orudis) C. Ketorolac (Toradol) D. Naproxyn (Naprosyn)

C. Ketorolac (Toradol)

The nurse is performing an admission assessment on an adolescent who reports taking extra strength acetaminophen (Tylenol) regularly to treat daily headaches. The nurse will notify the patient provider and discuss an order for A. A selective serotonin receptor agonist (SSRA) B. Hydrocodone with acetaminophen for headache pain C. Liver enzyme test D. Serum glucose testing

C. Liver enzyme test

The nurse is creating a pain management plan for a client with a previous history of substance abuse. Which of the following should be included in this plan? a.) Ask the physician to prescribe short-acting analgesics. b.) Ask the physician to prescribe a medication similar to the one the client abused. c.) Ask the physician to prescribe analgesics for the oral route. d.) Keep a dose of Narcan at the bedside.

C.) Ask the physician to prescribe analgesics for the oral route. Extended-release and long-acting analgesics are recommended for clients with a history of abuse. Specific interventions should: - Avoid analgesics similar to the abused drug - Utilize long-acting analgesics, - Avoid Narcan - Administer medications through the oral route

Cyclooxygenase (COX) enzyme

Coverts arachidonic acid into prostaglandins; Has two enzyme forms · COX-1: protects stomach lining and regulates blood platelets · COX-2: conversion to prostaglandins - triggers inflammation and pain

A client is receiving an antipsychotic agent. Which laboratory result is of most concern? a. Serum sodium level of 138 mEq/L b. Blood glucose level of 100 mg/dl c. White blood cell count of 6000/mm3 d. Serum medication level below normal limits

D ~ A serum medication level below normal limits is a concern because subtherapeutic levels may allow for breakthrough psychotic symptoms.

A patient comes to the mental health clinic for a regular appointment. The patient tells the nurse he has been taking oral fluoxetine (Prozac) 20 mg daily for the past 3 weeks and that he has lost 3 pounds during that time due to a loss of appetite. What action should the nurse take? A) Teaching the patient about healthy eating to maintain weight B) Congratulating the patient on his weight loss and commenting how well he looks C) Encouraging the patient to increase fluid intake to avoid further weight loss D) Reassuring the patient that a decrease in weight is a common adverse effect with this medication

D ~ Adverse effects of fluoxetine include anorexia and weight loss. Although teaching about healthy eating is a good idea, it is more important to teach the patient how to take the medication in a way that will reduce adverse effects as well as how to optimize healthy calories to maintain weight. The patient should increase caloric intake, not just fluid intake. The patient should continue the medication to see whether therapeutic effects are obtained and adjust nutritional intake if necessary. More information about the patients baseline weight is needed before congratulating the patient because a patient who is already too thin would not appreciate the nurses comment.

A patient receives a new prescription for fluvoxamine (Luvox). What will the nurse instruct the patient about taking the medication? A) Take medication after eating breakfast. B) Take medication with at least 8 ounces of liquid. C) The dosage may need to be increased if the patient is not feeling better in 2 weeks. D) The medication should be taken once a day before bedtime.

D ~ Fluvoxamine is a selective serotonin reuptake inhibitor that should be taken once a day before bedtime. The medication does not require 8 ounces of fluid for absorption. It should be taken for at least 4 weeks before a therapeutic effect is noted.

For what purpose would the nurse choose to administer a hypnotic instead of another classification of antianxiety drug? A) Treating insomnia B) Treating seizure disorder C) Treating panic attach D) Treating confusion and agitation

D ~ Hypnotics are used to help people fall asleep by causing sedation. Drugs that are effective hypnotics act on the reticular activating system (RAS) and block the brains response to incoming stimuli. Hypnotics would not be the most effective drugs to treat seizure disorders, panic attack, or confusion with agitation.

The nurse is preparing a dose of Mellaril. What is the highest priority intervention for the nurse while preparing the dose? a. Draw up the dosage of the liquid in an oral syringe. b. Use a 21-gauge needle to administer the injection. c. Start a new IV site before administering the drug. d. Avoid spilling the liquid on exposed skin.

D ~ If Mellaril is allowed to come in contact with exposed skin, contact dermatitis can result.

An older adult African American patient comes to the clinic and is diagnosed with generalized anxiety disorder (GAD). The physician orders oral flurazepam 30 mg. What is the nurses priority action? A) Teach the patient about the prescribed medication. B) Administer the first dose of medication. C) Tell patient to take first dosage after driving home. D) Talk to the physician about the dosage.

D ~ If an anxiolytic or hypnotic agent is the drug of choice for an African American patient, the smallest possible dose should be used, and the patient should be monitored very closely during the first week of treatment. Dosage adjustments are necessary to achieve the most effective dose with the fewest adverse effects. In addition, older adults also require careful titration of dosage. Older patients may be more susceptible to the adverse effects of these drugs, from unanticipated central nervous system (CNS) adverse effects including increased sedation, dizziness, and even hallucinations. Dosages of all of these drugs should be reduced and the patient should be monitored very closely for toxic effects and to provide safety measures if CNS effects do occur. As a result, the priority action is to talk to the physician about the dosage. The other actions may be appropriate after a proper dosage is ordered.

A client is ordered to receive diazepam (Valium). The nurse is teaching the client about her medication. Which information would be included in the teaching plan? a. The medication causes high levels of energy and activity. b. The medication is effective in aiding clients with suicidal ideations. c. The medication may be taken concurrently with other benzodiazepines. d. The client may develop tolerance after prolonged use.

D ~ Patients may become tolerant to Valium.

A 12-year-old patient is hospitalized with severe depression. The patient has been taking a selective serotonin reuptake inhibitor (SSRI). What is the priority nursing action for the patient? A) Monitor food intake for levels of tyramine. B) Assess for weight loss and difficulty sleeping. C) Monitor the patient for severe headaches. D) Implement suicide precautions.

D ~ Recent studies have linked the incidence of suicide attempts to the use of SSRIs in pediatric patients (see box 21.3 Focus on the Evidence). The priority concern for the nurse would be safety for the patient. Severe headache and reactions to tyramine-containing foods are associated with monoamine oxidase therapy. Weight loss and difficulty sleeping are of a lower priority concern than the patients safety.

What is the physiological action of tricyclic antidepressants (TCAs)? A) Inhibiting monoamine oxidase inhibitors that break down norepinephrine B) Inhibiting nerve activity, which prevents over excitability or stimulation C) Blocking the reuptake of serotonin, which increases the levels of norepinephrine D) Inhibiting reuptake of norepinephrine and serotonin

D ~ TCAs inhibit presynaptic reuptake of norepinephrine and serotonin, which cause an accumulation of the neurotransmitters that is thought to create the antidepressant effect. Monoamine oxidase inhibitors irreversibly inhibit monoamine oxidase that breaks down norepinephrine and serotonin. Selective serotonin reuptake inhibitors block the reuptake of serotonin; gamma-aminobutyric acid inhibits nerve activity.

A nurse is caring for a 9-year-old patient and has received an order for diazepam (Valium) 10 mg given orally q.i.d. What is the nurses priority action? A) Perform hand hygiene and prepare the drug. B) Send the order to the hospital pharmacy. C) Determine when to administer the first dose. D) Call the physician and question the order.

D ~ The first action of the nurse would be to call the physician and question the order. The normal oral dosage for a pediatric patient is 1 to 2.5 mg t.i.d. or q.i.d. The ordered dose would be unsafe for this patient. If the dosage was changed and the correct amount administered, the nurse would order the medication from the pharmacy if necessary and determine what time to start the medication. She would then wash her hands in preparation for administering the medication, but not until obtaining an appropriate dosage of medication.

A patient is being discharged home from the hospital after receiving treatment for pneumonia. The patient is going home and continuing to take the same drugs he or she was taking before he or she was hospitalized. These drugs include an antianxiety medication and a medication for insomnia. The home care nurse is following this patient. On the initial visit what is the nurses priority teaching point? A) The names and purposes of medications prescribed B) How to contact the provider if needed C) The importance of taking medications for insomnia only occasionally D) Warning signs that may indicate serious adverse effects

D ~ The home care nurse should provide thorough patient teaching, with a priority teaching point being the warning signs the patient may experience that indicate a serious adverse effect. Although this may have been discussed by the discharging nurse in the hospital, this is essential information for the patient to thoroughly understand. By the time the home care nurse visits, the patient should already have filled the prescriptions and know the names and purposes of the medications prescribed from the hospital nurse but it is a good idea to review this information, although it is not a priority. Medications for insomnia should be taken as prescribed. The patient should have received the providers contact information when leaving the hospital but the home care nurse may need to review this, even though it is not the priority teaching point.

The nurse is caring for a patient who has not been able to sleep. The physician orders a barbiturate medication for this patient. What adverse effect should the nurse teach the patient about? A) Double vision B) Paranoia C) Tinnitus D) Thinking abnormalities

D ~ The most common adverse effects are related to general central nervous system (CNS) depression. CNS effects may include drowsiness, somnolence, lethargy, ataxia, vertigo, a resembling a hangover, thinking abnormalities, paradoxical excitement, anxiety, and hallucinations. Barbiturate drugs generally do not cause double vision, paranoia, or tinnitus.

What is a common side effect for which the nurse must monitor during administration of both phenothiazine and non-phenothiazine medications? a. Hypertension b. Renal failure c. Increase in number of white blood cells d. Extrapyramidal symptoms

D ~ These medications are known for their extrapyramidal symptoms.

A patient presents at the emergency department with respiratory depression and excessive sedation. The family tells the nurse that the patient has been taking medication throughout the evening and gives the nurse an almost empty bottle of benzodiazepines. What other adverse effects would the nurse assess this patient for? A) Seizures B) Tachycardia C) Headache D) Coma

D ~ Toxic effects of benzodiazepines include excessive sedation, respiratory depression, and coma. Flumazenil (Anexate) is a specific antidote that competes with benzodiazepines for benzodiazepine receptors and reverses toxicity. Seizures, tachycardia, and headache would not normally be associated with benzodiazepine toxicity.

A 72-year-old patient presents at the emergency department with respiratory depression and excessive sedation. The family tells the nurse that the patient has been taking medication throughout the evening. The nurse suspects benzodiazepine overdose and would expect what drug to be ordered? A) Valium B) Phenergan C) Hydroxyzine D) Flumazenil

D ~ Toxic effects of benzodiazepines include excessive sedation, respiratory depression, and coma. Flumazenil is an antidote for the benzodiazepines. Hydroxyzine is an antihistamine with anticholinergic effects and would not be appropriate for this patient. Valium would enhance the effects of benzodiazepines. Phenergan is not indicated for this patient; it is similar in actions to hydroxyzine.

A patient takes oxycodone (OxyContin), 40 mg PO twice daily, for the management of chronic pain. Which intervention should be added to the plan of care to minimize the gastrointestinal adverse effects? A) Take an antacid with each dose. B) Eat foods high in lactobacilli. C) Take the medication on an empty stomach. D) Increase fluid and fiber in the diet.

D) Increase fluid and fiber in the diet. Narcotic analgesics reduce intestinal motility, leading to constipation. Increasing fluid and fiber in the diet can help manage this adverse effect.

The nurse administers nalbuphine (Nubian) to a patient who is experiencing severe pain. Which statement by the patient indicates a need for further teaching about this drug A. I may experience unusual dreams while taking this medication B. I may need to use a laxative when taking this drug C. I should ask for assistance when getting out of bed D. I should expect to have more frequent urination

D. I should expect to have more frequent urination

One hour after receiving intravenous morphine sulfate, a picture reports generalized itching. The nurse assesses the patient notes clear breath sounds, respirations are 14 breaths per minute, a heart rate of 68 bpm, and a blood pressure of 110/70 mm Hg. Which action will the nurse take A. Administered naloxone to reverse opiate overdose B. Have resuscitation equipment available at the bedside C. Prepare an epinephrine injection in case of an anaphylactic reaction D. Reassure the patient that this is a common side effect of this drug

D. Reassure the patient that this is a common side effect of this drug

The nurse is evaluating A patient two hours after giving a Joseph 30 mg of codeine with acetaminophen for postoperative pain after abdominal surgery. The patient reports of pain level seven on a scale of 1 to 10. The nurse notes a heart rate of 110 bpm, respiratory rate of 20 breaths per minute, and a blood pressure of 180/90 mm Hg. Which action will the nurse take? A. Administer the next dose of Codeine one hour early B. Ask the provider of the codeine dose can be increased C. Contact the provider to ask if a dose of ibuprofen maybe given now D. Request an order for oxycodone with acetaminophen (Percocet)

D. Request an order for oxycodone with acetaminophen (Percocet)

The emergency department nurse is caring for a patient who has received morphine sulfate for severe pain following an injury. The nurse performs a drug history and learned that the patient take St. John's wort for symptoms of depression. The nurse will reserve the patient closely for an increase in which opioid adverse effect A. Constipation B. Pruritis C. Respiratory depression D. Sedation

D. Sedation

A client in the ICU who sustained a traumatic abdominal injury 1 week ago continues to complain of severe pain. The nurse notes his vital signs are normal. Which of the following would be appropriate for the nurse to do? a.) Encourage early return to ambulation. b.) Offer nonnarcotic analgesics for pain. c.) Utilize distraction d.) Provide the client with pain medication.

D.) Provide the client with pain medication.

NSAID use in older adults

Drug interactions more common; Greater incidence of GI distress/ulceration; Reduced dose decreases risk of side effects; Encourage increased fluid intake

Allopurinol (ANTI-GOUT)

Indication: given for gouty patients with renal impairment, given prophylactically Acton: lowers serum uric acid level, prevent gouty attacks Pt care interventions: · Record intake and output · Monitor lab tests for renal and hepatic function · Instruct pt to increase fluid intake to assist in excretion of uric acid and drugs · Advise pt to have yearly eye exams, prolonged use can result in visual changes

Gout

Inflammatory disease of joints, tendons, and other tissues; Defect in purine metabolism leads to uric acid accumulation; Purine containing foods: organ meats, sardines, salmon, gravy, liver, meat soups and alcohol (beer)

Delayed phase of inflammation

Leukocytes infiltrate inflamed tissue

Uricosurics (anti-gout drugs)

Probenecid -Action: blocks reabsorption of uric acid which promotes its excretion, alleviates chronic gout -Side effects: flushed skin, headache, sore gums, GI distress, kidney stones, sever blood dyscrasias (rare) -Can be given with colchicine, but it must be given before probenecid -Sulfinpyrazone is more potent than probenecid - take with food or antacids -Do not use in attacks of gout

Hydromorphone (Dilaudid)

Use: 6 times more potent than morphine Withdrawal syndrome: caused by physical dependence · Irritability, diaphoresis, restlessness, muscle twitching, tachycardia, increased BP

A patient has begun taking buspirone hydrochloride (BuSpar) 7.5 mg twice daily to treat acute anxiety and calls 1 week later to report little change in symptoms. What will the nurse tell the patient? a. "Therapeutic effects may not be evident for another week." b. "The provider may need to increase the dose to 15 mg twice daily." c. "Notify the provider and request an order for another anxiolytic." d. "Stop taking the drug and notify the provider that it doesn't work."

a. "Therapeutic effects may not be evident for another week."

The nurse is assessing a young adult patient with schizophrenia who recently began taking fluphenazine (Prolixin). The patient is exhibiting spasms of facial muscles along with grimacing, and the nurse notes upward eye movements. The nurse suspects which side effect? a. Acute dystonia b. Akathisia c. Pseudoparkinsonism d. Tardive dyskinesia

a. Acute dystonia

A patient has been taking sertraline (Zoloft) 20 mg/mL oral concentrate, 1 mL daily for several weeks and reports being unable to sleep well. What will the nurse do next? a. Ask the patient what time of day the medication is taken. b. Counsel the patient to take the medication at bedtime. c. Recommend asking the provider about weekly dosing. d. Suggest that the patient request a lower dose.

a. Ask the patient what time of day the medication is taken.

A nurse performs a medication history on a newly admitted patient. The patient reports taking amitriptyline (Elavil) 75 mg at bedtime for 6 weeks to treat depression. The patient reports having continued fatigue, lack of energy, and depressed mood. The nurse will contact the provider to discuss which intervention? a. Beginning to taper the amitriptyline b. Changing to a morning dose schedule c. Giving the amitriptyline twice daily d. Increasing the dose of amitriptyline

a. Beginning to taper the amitriptyline

The parent of a young adult who has schizophrenia is concerned that the patient spits out pills that are given. The nurse will suggest contacting the patient's provider to discuss which intervention? a. Changing to a liquid form of the mediation b. Providing a home health nurse to supervise medication administration c. Teaching the patient the importance of taking the medication d. Weekly intramuscular injections of the medication

a. Changing to a liquid form of the mediation

A patient who has a major depressive disorder has been taking fluoxetine (Prozac) 20 mg daily for 3 months and reports improved mood, less fatigue, and an increased ability to concentrate. The patient's side effects have diminished. What will the nurse counsel this patient to discuss with the provider? a. Changing to once-weekly dosing b. Decreasing the dose to 10 mg daily c. Discontinuing the medication d. Increasing the dose to 30 mg daily

a. Changing to once-weekly dosing

A patient who has recently begun taking lithium (Lithibid) calls the clinic to report nausea, vomiting, anorexia, and drowsiness. What will the nurse do next? a. Contact the provider to obtain an order for a serum lithium level. b. Reassure the patient that these symptoms are common and transient. c. Tell the patient that the lithium dose is probably too low. d. Tell the patient to stop taking the medication immediately.

a. Contact the provider to obtain an order for a serum lithium level.

A patient arrives in the emergency department with dehydration. The patient reports taking fluphenazine (Prolixin) to treat schizophrenia. The nurse notes rigid muscles and an altered mental status. The patient has a temperature of 103.6° F, a heart rate of 98 beats per minute, and a blood pressure of 90/58 mm Hg. The nurse will anticipate administering which medication? a. Dantrolene (Dantrium) b. Haloperidol (Haldol) c. Propranolol (Inderal) d. Tetrabenazine (Xenazine)

a. Dantrolene (Dantrium)

A resident of the nursing home has quite severe arthritis. When administering an analgesic to this elderly resident, the nurse should: a. Give the medication before the activity session in the day room b. Give the medication when the resident states the pain is at 6 or higher on a 1-10 pain scale c. Give the pain medication at mealtime d. Make sure that the medication is not a narcotic

a. Give the medication before the activity session in the day room.

A patient who is taking amitriptyline (Elavil) reports constipation and dry mouth. The nurse will give the patient which instruction? a. Increase fluid intake. b. Notify the provider. c. Request another antidepressant. d. Stop taking the medication immediately.

a. Increase fluid intake.

A patient who has had a loss of interest in most activities, weight loss, and insomnia is diagnosed with a major depressive disorder and will begin taking fluoxetine (Prozac) daily. The patient asks about the weekly dosing that a family member follows. What will the nurse tell the patient about a weekly dosing regimen? a. It can be used after daily maintenance dosing proves effective and safe. b. It is used after a trial of tricyclic antidepressant medication fails. c. It is not effective for this type of depression and its symptoms. d. It will cause more adverse effects than daily dosing regimens.

a. It can be used after daily maintenance dosing proves effective and safe.

The nurse is preparing to administer paroxetine HCl (Paxil) to a 70-year-old patient. The nurse understands that this patient may require a. a decreased dose. b. an increased dose. c. every other day dosing. d. more frequent dosing.

a. a decreased dose.

Naloxone (Narcan) is administered to a client with severe respiratory depression and suspected drug overdose. After 20 minutes, the client remains unresponsive. The most likely explanation for this is: a.) The client did not use an opioid drug. b.) The dose of naloxone was inadequate. c.) The client is resistant to this drug. d.) The drug overdose is irreversible.

a.) The client did not use an opioid drug. If opioid antagonists (Naloxone) fail to reverse symptoms of respiratory depression quickly, the overdose was likely due to a non-opioid substance.

The nurse is teaching a patient who will be discharged home on a typical antipsychotic medication to treat schizophrenia. Which statement by the patient indicates a need for further teaching? a. "I should not drink alcohol while taking this medication." b. "I should use a heating pad to treat muscle spasms while taking this medication." c. "I should use sunscreen while taking this medication." d. "I will need frequent blood tests while taking this medication."

b. "I should use a heating pad to treat muscle spasms while taking this medication."

A patient who is about to begin taking the atypical antipsychotic medication clozapine (Clozaril) is concerned about side effects. What information will the nurse include when teaching the patient about this medication? a. "You are more likely to experience dry mouth, constipation, and urinary retention." b. "You may experience weight gain, drowsiness, and headaches." c. "You will not experience extrapyramidal side effects with this medication." d. "You will not need frequent lab work while taking this medication."

b. "You may experience weight gain, drowsiness, and headaches."

A patient who has a history of migraine headaches is diagnosed with bipolar disorder. The nurse might expect the provider to order which medication for this patient? a. Carbamazapine (Tegretol) b. Divalproex (Valproate) c. Lamotrigine (Lamictal) d. Lithium citrate (Eskalith)

b. Divalproex (Valproate)

A patient is brought to the emergency department with decreased respirations and somnolence. The nurse notes a heart rate of 60 beats per minute and a blood pressure of 80/58 mm Hg. The patient is known to take alprazolam (Xanax) to treat anxiety. Which medication will the nurse anticipate the provider to order? a.Benztropine (Cogentin) b. Flumazenil (Romazicon) c. Lorazepam (Ativan) d. Propranolol (Inderal)

b. Flumazenil (Romazicon)

A patient who has been diagnosed with a major depression disorder has been ordered to take doxepin (Sinequan). The nurse will contact the provider if the patient's medical history reveals a history of which condition? a. Asthma b. Glaucoma c. Hypertension d. Hypoglycemia

b. Glaucoma

The nurse is preparing to administer loxapine (Loxitane) 50 mg to a patient who has schizophrenia. The patient has been taking this medication twice daily for 15 months. The nurse notes smacking lip movements and involuntary movements of all extremities. Which action by the nurse is correct? a. Administer the medication as ordered to treat these symptoms of psychosis. b. Hold the dose and notify the provider of these medication adverse effects. c. Request an order for an anticholinergic medication such as benztropine (Cogentin). d. Suggest that the provider increase the dose to 125 mg twice daily.

b. Hold the dose and notify the provider of these medication adverse effects.

The nurse is preparing to administer a dose of lithium (Lithibid) to a patient who has been taking the drug as maintenance therapy to treat bipolar disorder. The nurse assesses the patient and notes tremors and confusion. The patient's latest serum lithium level was 2 mEq/L. Which action will the nurse take? a. Administer the dose. b. Hold the dose and notify the provider. c. Request an order for a higher dose. d. Request an order for a lower dose.

b. Hold the dose and notify the provider.

A patient who takes loxapine (Loxitane) to treat schizophrenia is noted to be restless and fidgety and is pacing around the room. The nurse caring for this patient will perform which action? a. Contact the provider to discuss changing to benztropine (Cogentin). b. Notify the provider of these symptoms and request an order for lorazepam (Ativan). c. Question the patient about adherence to the drug regimen. d. Recognize these signs of a serious adverse drug reaction and notify the provider.

b. Notify the provider of these symptoms and request an order for lorazepam (Ativan).

A patient has been taking risperidone (Risperdal) for 2 weeks. The patient reports drowsiness and headache. What will the nurse do? a. Counsel the patient to request changing to aripiprazole (Abilify). b. Reassure the patient that these are common side effects of the medication. c. Suggest that the patient have serum glucose testing. d. Suggest that these may be signs of agranulocytosis.

b. Reassure the patient that these are common side effects of the medication.

A male patient has been taking venlafaxine (Effexor) 37.5 mg daily for 2 weeks and reports ejaculation dysfunction and urinary retention. What action will the nurse take? a. Contact the provider to discuss decreasing the dose. b. Reassure the patient that these are common side effects. c. Report potential serious adverse effects to the provider. d. Withhold the dose until the provider is notified.

b. Reassure the patient that these are common side effects.

A patient has been taking paroxetine (Paxil) 20 mg per day for 2 weeks and reports headaches, nervousness, and poor appetite. Which action will the nurse take? a. Counsel the patient to take the medication with food. b. Reassure the patient that these side effects will decrease over time. c. Suggest that the patient discuss a lower dose with the provider. d. Tell the patient to stop taking the drug and contact the provider.

b. Reassure the patient that these side effects will decrease over time.

A patient who is taking chlorpromazine calls the clinic to report having reddish-brown urine. What action will the nurse take? a. Notify the provider and request orders for creatinine clearance and BUN levels. b. Reassure the patient that this is a harmless side effect of this medication. c. Tell the patient to come to the clinic for a urinalysis. d. Tell the patient to discard any drug on hand and request a new prescription.

b. Reassure the patient that this is a harmless side effect of this medication.

Two days after surgery, an elderly client refuses a PRN dose of analgesic dose for fear of becoming "hooked." The nurse should respond by stating that: a. It is impossible to become hooked on PRN narcotics. b. Short-term use of narcotics is not likely to cause a person to become dependent on them. c. Side effects that occur in the elderly mean that medications will be discontinued as soon as possible. d. The elderly are least likely to become dependent on narcotics.

b. Short-term use of narcotics is not likely to cause a person to become dependent on them.

A patient who has been taking a monoamine oxidase (MAO) inhibitor for several months will begin taking amoxapine (Asendin) instead of the MAO inhibitor. The nurse will counsel the patient to begin taking the amoxapine a. along with the MAO inhibitor for several months. b. at least 14 days after discontinuing the MAO inhibitor. c. the day after the last dose of the MAO inhibitor. d. while withdrawing the MAO inhibitor over several weeks.

b. at least 14 days after discontinuing the MAO inhibitor.

Nursing intervention for a client receiving opioid analgesics over an extended period of time should include: a.) Referring the client to a drug treatment center. b.) Encouraging increased fluids and fiber in the diet. c.) Monitoring for G.I. bleeding. d.0 Teaching the client to take her own blood pressure.

b.) Encouraging increased fluids and fiber in the diet. Opioids suppress intestinal contractility, increase anal sphincter tone, and inhibit fluids into the intestines, which can lead to constipation. There is nothing to indicate the drug is related to addiction problems. Opioids do not cause GI bleeding.

A patient who is taking fluphenazine (Prolixin) to treat psychosis is experiencing symptoms of acute dystonia. While performing a medication history, the nurse learns that the patient takes herbal medications. Which herbal supplement would be of concern? a. Ginkgo b. Ginseng c. Kava kava d. St. John's wort

c. Kava kava

A patient arrives in the emergency department complaining of difficulty breathing, dizziness, sweating, and heart palpitations. The patient reports having had similar episodes previously. The nurse will expect the provider to order which medication? a. Flumazenil (Romazicon) b. Haloperidol (Haldol) c. Lorazepam (Ativan) d. Propranolol (Inderal)

c. Lorazepam (Ativan)

A patient who takes a monoamine oxidase (MAO) inhibitor asks the nurse about taking over-the-counter medications to treat cold symptoms. Which medication will the nurse counsel the patient to avoid while taking an MAO inhibitor? a. Diphenhydramine b. Guaifenesin c. Pseudoephedrine d. Saline nasal spray

c. Pseudoephedrine

An elderly client had abdominal surgery six hours earlier. When the nurse asks the client about pain, the client responds that there is none. The best intervention on the part of the nurse is: a. Administer a PRN dose of IV pain medication as ordered. b. Assist the client into a sitting position in preparation for ambulation. c. Question the client further about discomfort to assess the meaning of pain. d. Assess the abdominal dressing and consult the surgeon about findings.

c. Question the client further about discomfort to assess the meaning of pain.

A family member of a patient who has been taking fluphenazine (Prolixin) for 3 months calls to report that the patient is exhibiting agitation and restlessness. The nurse learns that the patient's delusional thinking and hallucinations have stopped since taking the medication. The nurse will perform which action? a. Reassure the family member that tolerance to these side effects will subside over time. b. Remind the family member that complete drug effects may not occur for several more weeks. c. Suggest that the family member contact the provider to discuss an order for a benzodiazepine. d. Tell the family member to withhold the medication

c. Suggest that the family member contact the provider to discuss an order for a benzodiazepine.

A client who incurred an arm injury describes his pain as "sharp and localized to the lower arm." The nurse recognizes that this type of pain would be relieved best by administration of which type of medication? (Select all that apply) a.) Muscle relaxant b.) Acetaminophen c.) Narcotic analgesics d.) NSAIDs

c.) Narcotic analgesics d.) NSAIDs Injury to tissues produces nociceptor pain, which usually responds to conventional analgesic pain medications such as opiates or NSAIDS.

The client admitted with hepatitis B is prescribed Vicodin 2 tabs for treatment of pain. The appropriate nursing action is to: a.) Administer the drug as ordered. b.) Administer one tablet only. c.) Question the physician about the order. d.) Hold the drug until the physician arrives.

c.) Question the physician about the order. Vicodin is a combination drug of hydrocodone and acetaminophen. Acetaminophen can be hepatotoxic, and is contraindicated in liver disease.

The nurse is preparing to administer intramuscular haloperidol (Haldol) to a patient who has schizophrenia. What action will the nurse perform? a. Massage the site after injecting the medication to ensure complete absorption. b. Teach the patient to return every week to receive medication doses. c. Use a small bore needle when injecting the medication. d. Use the Z-track method and inject the medication into deep muscle tissue.

d. Use the Z-track method and inject the medication into deep muscle tissue.

The nurse provides teaching for a patient who will begin taking lithium (Lithibid). Which statement by the patient indicates understanding of the teaching? a. "I may drink tea or cola but not coffee." b. "I may stop taking the drug when mania symptoms subside." c. "I should consume a sodium-restricted diet." d. "I should drink 2 to 3 liters of fluid each day."

d. "I should drink 2 to 3 liters of fluid each day."

The nurse is teaching a patient about taking an anxiolytic agent to treat grief-related anxiety. Which statement by the patient indicates understanding of the teaching? a. "I may have wine with dinner to help with relaxation." b. "I may need to take this medication for up to a year." c. "I may stop taking the medication when my symptoms go away." d. "I should try psychotherapy or a support group in addition to the medication."

d. "I should try psychotherapy or a support group in addition to the medication."

The nurse is teaching a patient who will begin taking doxepin (Sinequan) to treat depression. Which statement by the patient indicates a need for further teaching? a. "I should expect results within 2 to 4 weeks." b. "I should increase fluids and fiber while taking this medication." c. "I should take care when rising from a sitting to standing position." d. "I will take the medication in the morning before breakfast."

d. "I will take the medication in the morning before breakfast."

The nurse assesses a patient who is taking lithium (Lithibid) and notes a large output of clear, dilute urine. The nurse suspects which cause for this finding? a. Cardiovascular complications b. Expected lithium side effects c. Increased mania d. Lithium toxicity

d. Lithium toxicity

The safest narcotic choice for an elderly client with acute pain is: a. Meperidine (Demerol) b. Oxycodone c. Fentanyl transdermal patch d. Morphine sulfate

d. Morphine sulfate. Rationale: Morphine is the "gold standard" of narcotics for acute pain. The other choices are incorrect.

A patient who has been diagnosed with social anxiety disorder will begin taking venlafaxine (Effexor). The nurse who performs a medication and dietary history will be concerned about ingestion of which substance or drug? a. Coffee b. Grapefruit juice c. Oral hypoglycemic drug d. St. John's wort

d. St. John's wort

A patient who has been diagnosed with depression asks why the provider has not ordered a monoamine oxidase (MAO) inhibitor to treat the disorder. The nurse will explain to the patient that MAO inhibitors a. are more expensive than other antidepressants. b. are no longer approved for treating depression. c. can cause profound hypotension. d. require strict dietary restrictions.

d. require strict dietary restrictions.

The nurse is caring for a patient who has begun taking chlorpromazine HCl (Thorazine) 75 mg BID to treat schizophrenia. A family member tells the nurse that the patient's agitation, hallucinations, and delusional symptoms have improved, but the patient continues to withdraw from social interaction and won't bathe unless reminded to do so. The nurse will tell the family member that a. all symptoms will eventually resolve over time with this medication. b. the patient may need an increased dose of the antipsychotic medication. c. these results may indicate that the patient does not have schizophrenia. d. they should consider discussing adding another medication.

d. they should consider discussing adding another medication.

The nurse is performing a medication history on a patient who reports taking lorazepam (Ativan) for the past 6 months to treat an anxiety disorder. The patient states that the medication is not working as well as previously. The nurse will a. contact the provider to discuss changing to another benzodiazepine. b. notify the provider and discuss increasing the dose of lorazepam. c. suspect worsening of the anxiety disorder. d. understand that the patient has developed tolerance to this drug.

d. understand that the patient has developed tolerance to this drug.

The client informs the nurse that he has experienced pain in the lower extremities for the past eight months. The nurse recognizes that this pain is classified as: a.) Moderate. b.) Severe. c.) Acute. d.) Chronic.

d.) Chronic. Chronic pain persists longer than six months.

The nurse administers morphine sulfate 4 mg IV to a client for treatment of severe pain. Which of the following assessments requires immediate nursing interventions? a.) Blood pressure 110/70 b.) The client is drowsy. c.) Pain is unrelieved in 15 minutes. d.) Respiratory rate 10/minute

d.) Respiratory rate 10/minute Opioids activate mu and kappa receptors that can cause profound respiratory depression. Respiratory rate should remain above 12. The BP is not significantly low. Drowsiness is an expected effect of morphine. Unrelieved pain warrants further assessment, but not as immediately as do decreased respirations.

Acetaminophen (Tylenol)

o Action: inhibits prostaglandin synthesis o Use: muscular aches and pains, fever o Side effects: Rash, headache, insomnia, Hepatotoxicity, renal failure, Thrombocytopenia, Hemolytic anemia, Agranulocytosis. Leukopenia, neutropenia o Check liver enzymes

Opioid analgesics

o Controlled substances o Potential for abuse and psychological dependence o Use: moderate-severe pain o Action: § Acts on CNS: leads to analgesia and sedation; Suppress respiration and coughing by acting on resp and cough centers in medulla (morphine, codeine) o Opioids have to isomers: Levo > analgesic effect only (physical dependence); Levo + dextro > anti-tissue response · Contraindications: increase cardiac workload · Interactions: o CNS depressants (barbiturates, phenobarbital, benzodiazepines, EtOH) o Anticholinergics, antihistamines, tricyclic antidepressant o MAOIs (hyperpyrexia, seizures) o Antihypertensives · Education: o Withhold if RR < 12 o Have naloxone ready o Infuse IV slowly over 4-5 min

Aspirin

o Do NOT take with other NSAIDs o Avoid during last trimester of pregnancy o Do NOT give to children with flu or virus symptoms; may lead to Reye Syndrome (Reye Syndrome à acute encephalopathy and fatty infiltrations of liver and possibly the pancreas, heart, kidney, and spleen, and lymph nodes) o Relieve pain by inhibiting COX enzymes o Side effects: dizziness, drowsiness, headache, tinnitus, ulceration, bleeding, GI distress, thrombocytopenia, leukopenia, agranulocytosis, hemolytic anemia, hepatotoxicity, Reye o Hypersensitivity/overdose > tinnitus, vertigo, bronchospasm o Pt care notes: -Observe for bleeding -Do not take with alcohol or warfarin -Discontinue ~7 days before surgery

Adverse reactions of antipsychotics

o Extrapyramidal syndrome: Stooped posture, masklike facies, rigidity, tremors at rest (pseudo-parkinism); Shuffling gait, pill-rolling motion of hands, bradykinesia o Acute dystonia: Muscle spasm of face, tongue, neck and back; Abnormal or involuntary upward eye movement, laryngeal spasms/impaired respirations o Akathisia: Pt has trouble standing still o Tardive dyskinesia: Rolling of tongue, smacking movements of lips, chewing motion; Involuntary movement of body and extremities o Neuroleptic malignant syndrome: Rare but potentially fatal · Excess agitation, altered mental status · Exhaustion, dehydration · Muscle rigidity, hyperthermia, profuse diaphoresis · BP fluctuations, tachycardia, dysrhythmias · Seizures, rhabdomyolysis · Respiratory and renal failure, coma

Corticosteroids

o Prednisone, prednisolone, dexamethasone -Action: control inflammation by suppressing components of inflammatory response at injured site · Long half life (over 24 hrs) § Use: arthritic flare up · Not drug of choice for arthritis because numerous side effects -Discontinuation: taper off over 5-10 days

disease-modifying antirheumatic drugs (DMARDs)

o Use: Alleviate symptoms of RA when other treatments fail; Osteoarthritis, ankylosing spondylitis; Psoriatic arthritis, severe psoriasis; Crohn's, UC o Side effects/adverse rxns: Headache, fever, chills, insomnia, fatigue, peripheral edema; Oral ulcerations, nasopharyngitis, infection; GI distress, elevated hepatic enzymes, weakness; Hypotension/hypertension, hypercholesteremia; Aplastic anemia, leukopenia, neutropenia, thrombocytopenia

Salicylates (First generation NSAIDs/Aspirin)

· Action: anti-inflammatory, antiplatelet, antipyretic effects · Therapeutic serum level: 15-30 mg/dL · Toxic serum level: > 30mg/dL (mild), >50 mg/dL (severe) · Drug - Lab - Food interactions: o Increased bleeding with anticoagulants and other NSAIDs o Risk for hypoglycemia with oral antidiabetics o Increased gastric ulcer risk with glucocorticoids o Decreased effects of ACE inhibitors, loop diuretics, probenecid o Effects are decreased by corticosteroids o Increase PT/INR, bleeding time, uric acid o Decrease cholesterol, T3, T4 levels o Prunes, raisins, licorice o Certain spices (curry, and paprika)

Antipsychotics

· Action: dopamine antagonists - blocks dopamine receptors in brain · Major categories: o Typical: Phenothiazines · Block norepinephrine causing sedative and hypotensive effects early in treatment Nonphenothizines: · Block only the neurotransmitter dopamine - Medication may take 6 + weeks to achieve full clinical effect - Monitor for hypoglycemia or other diabetes symptoms of DM o Atypical: Treats schizophrenia and other psychotic disorders in pt's not responding to or intolerant of typical Decreased side effects Often used as first line therapy

Antimalarials (mabs, DMARDs)

· Action: unclear (effect may take 4-12 weeks to become apparent) · Use: RA when other methods fail · Used with NSAIDs when arthritis not under control

Selective COX-2 inhibitors

· Celecoxib (Celebrex) · Action: selectively inhibits COX-2 enzyme without inhibition of COX-1 (which protects stomach lining, regulates platelets) · Use: osteoarthritis, RA, dysmenorrhea, ankylosing spondylitis, pain · Side effects: dizziness, headache, sinusitis, peripheral edema, renal dysfunction, hypertension, GI distress/bleeding

Proprionic Acid Derivatives

· Ibuprofen o Most widely used · Action: inhibits prostaglandin synthesis · Use: pain, osteoarthritis, RA · Side effects: o Drowsiness, dizziness, headache, confusion, o insomnia, dreams, blurred vision, edema o GI distress, bleeding, o Tinnitus, dysrhythmias, nephrotoxicity · Drug interactions: o Increased bleeding with warfarin o Increased effects with phenytoin, sulfonamides, warfarin, cephalosporins o Decreased effect with aspirin · Pt care interventions o Observe for bleeding (gums, petechiae, ecchymoses, stool) o Report GI discomfort o Advise to avoid alcohol when taking NSAIDs o Alert that some comp and alternative therapies may interact with NSAIDs and cause bleeding

Perchlorobenzoic acid

· Indomethacin, sulindac, tolmetin · Action: inhibits prostaglandin synthesis · Uses: RA, osteoarthritis, gouty arthritis · Side effects/adverse rxns: o Dizziness, headaches, weakness, GI distress, bleeding o Sodium and water retention, hypertension · Hard on stomach, taken with food

Immunomodulators (mabs, DMARDs)

· Infliximab (IV), adalimumab (SQ), leflunomide (oral) · Class: IL-1 receptor antagonists, tumor necrosis factor (TNF) · Use: RA, psoriatic arthritis, psoriasis, spondylitis, UC, Crohn's

Phenylacetic Acid Derivatives

· Ketorolac: injectable (short term), oral and extended · Diclofenac: extended relief and topical · Action: inhibits prostaglandin synthesis · Use: o RA, osteoarthritis, ankylosing spondylitis and pain o No antipyretic effect · Side effects/adverse rxns; o Dizziness, drowsiness, headache, weakness, stroke o GI distress, GI bleeding/perforation · Recommended for SHORT TERM pain management

Fenamates

· Meclofenamide sodium monohydrate, mefenamic acid · Action: inhibits prostaglandin synthesis · Use: osteoarthritis, RA, pain, dysmenorrhea · Side effects/adverse rxns: o Dizziness, headache, tinnitus, pruritis edema, GI distress/bleeding, renal dysfunction, elevated hepatic enzymes, stroke · Avoid if hx of peptic ulcer **

Oxicams

· Meloxicam, Piroxicam o For long term arthritic conditions o Advantage: long half-life, take daily · Action: inhibits prostaglandin synthesis · Use: RA, osteoarthritis · Side effects/adverse rxns: o Dizziness, headache, insomnia, edema, GI distress/bleeding, renal dysfunction, angioedema, elevated hepatic enzymes, stroke · Drugs well tolerated

Immunosuppressive agents (mabs, DMARDs)

· Methotrexate, cyclophosphamide, azathioprine · Action: suppress inflammatory process - primary use to stop cancer growth and proliferation · Use: refractory RA unresponsive to anti-inflammatory drugs


Conjuntos de estudio relacionados

Drugs and Society Test 3- ch. 6, 7, 8, 9

View Set

RN Targeted Medical Surgical Neurosensory and Musculoskeletal

View Set

hesi exit eaq respiratory questions

View Set

Psychology, Intro to: final exam, chapter 9-16

View Set

Chapter 6 - Economic Inequality, Wealth, and Poverty

View Set